Manipal Medical Manipal Medical Solved Paper-2006

  • question_answer
    Figure shows three points A, B and C in a region of uniform electric field\[\overrightarrow{E}\]. The line AB is perpendicular and BC is parallel to the field lines. Then which of the following holds good?

    A)  \[{{V}_{A}}={{V}_{B}}={{V}_{C}}\]

    B) \[{{V}_{A}}={{V}_{B}}>{{V}_{C}}\]

    C)  \[{{V}_{A}}={{V}_{B}}<{{V}_{C}}\]

    D)  \[{{V}_{A}}>{{V}_{B}}={{V}_{C}}\]

    Correct Answer: B

    Solution :

     Electric lines of force flow from higher potential to lower potential so, \[{{V}_{A}}={{V}_{B}}>{{V}_{C}}\] where\[{{V}_{A}}>{{V}_{B}}\]and\[{{V}_{C}}\]represent the electric potential at points A, B and C respectively.


You need to login to perform this action.
You will be redirected in 3 sec spinner